2016 PRITE Q/As

Ace your homework & exams now with Quizwiz!

The rate of synaptogenesis peaks at roughly which of the following times in human development? A. Second trimester of pregnancy B. Third trimester of pregnancy C. The first four years of life D. Between 5 and 10 years of life E. Between 15 and 20 years of life

C Neurosciences

Which of the following is characteristic of acetylcholine receptors found in the cerebral cortex? A. Blocked by curare B. Predominantly nicotinic C. Both inhibitory and excitatory D. Increased in Alzheimer disease E. Predominantly found in the substantia nigra

C Neurosciences

11. The National Institute of Mental Health (NIMH) was founded during a period of therapeutic optimism following which historical event? A. Discovery of chlorpromazine B. Invention of the polio vaccine C. Passing of the Civil Rights Acts D. Treatment of soldiers during World War II E. Establishment of the food and drug administration (FDA)

D

118. Which of the following is a first-line treatment for posttraumatic stress disorder (PTSD) in children and adolescents? A. Prazosin B. Propranolol C. Eseitalopram D. Cognitive-behavioral therapy (CBT) E. Interpersonal psychotherapy (IPT)

D

125. The parent of a 43 yr old patient died 5 yrs ago from pancreatic cancer. Four years ago, the patient noticed occasionally feeling uncomfortably full after eating large, fatty meals, and began fearing that this was an early sign of pancreatic cancer. The patient acknowledges that this is unlikely, but feels the need to frequently wiegh himself to be reassured that he is not losing weight. The patient had previously scheduled annual physicals, but now avoids seeing the doctor due to the fear of being diagnosed with cancer. There are no other psychiatric symptoms. Which of the following is this patients most likely diagnosis? A. Obsessive-compulsive disorder (OCD) B. Delusional disorder, somatic type C. Somatic symptom disorder D. lllness anxiety disorder E. Conversion disorder

D

129. Which of the following psycotherapeutic approaches to the treatment of substance use disorder are based on the prniciples of expressing empathy, developing discrepancy, avoiding argumentation, rolling with resistance and supporting self efficacy? A. Network therapy B. Twelve step facilitation C. Contingency management D. Motivational enhancement therapy E. Cognitive-behavioral therapy (CBT)

D

131. A 34-year-old patient with epilepsy is relatively seizure-free with antiepileptic medications but develops an episode of major depressive disorder. Addition of which of the following psychotropic medications is most likely to assist in treating the patient's depression? A. Amoxapine B. Aripiprazole C. Desipramine D. Sertraline E. Bupropion

D

136. An 80 -year-old patient with a history of hypertension and hypercholesterolemia develops the sudden onset of difficulty with speech and cannot understand written words. The patient retains the ability to write fluently, but is unable to comprehend what is written. A lesion including which of the following areas of the'"brain is likeiyto account for this syndrome? A. Thalamus B. Frontal lobe C. Angular gyrus D. Corpus callosum E. Supplementary motor cortex

D

139. Which of the following terms is used to describe the feeling of being outside of oneself or detached? A. Fugue B. Amnesia C. Derealization D. Depersonalization

D

140. Which of the following components of attention-deficit hyperactivity disorder (ADHD) is most likely to remain in adulthood? A. Hyperactivity B. Opposition C. Impulsivity D. Inattention

D

18. Which of the following is the best treatment for genito-pelvic pain/penetration disorder? A. Sildenafil B. Prostaglandin E1 C. Prohibition of sexual play D. Dilation with graduated sized vaginal dilators E. Constant reinforcement by partner during intercourse

D

Which of the following statements about the brain's default mode network is accurate? A. Is activated by external stimuli B. Localizes primarily to the non-dominant hemisphere C. Uses much less energy than goal-oriented brain activity D. Is involved in reprocessing previously experienced stimuli

D Neurosciences

A diagnosis of Huntington disease would be best confirmed by which of the following methods? A. Karyotyping B. Genetic linkage analysis C. Fluorescent in situ hybridization D. Trinucleotide expansion analysis E. Mitochondrial DNA insertion analysis

D Neurosciences

A patient presents with ptosis of the right eye, a dilated right pupil, double vision and left-sided hemiparesis without cognitive impairment. Which of the following brain regions is the likely lesion site? A. Basal ganglia B. Cerebral cortex C. Medulla D. Midbrain E. Pons

D Neurosciences

Among neurodevelopmental risk factors for aggression in male children, the evidence is strongest for a gene/environment interaction between maltreatment and genes for which of the following? A. Oxytocin B. Serotonin-transporter C. Dopamine-transporter D. Monoamine oxidase A E. Brain-derived neurotrophic factor

D Neurosciences

In which region of the brain does the morphology of neurons change the most during old age? A. Amygdala B. Hippocampus C. Perirhinal cortex D. Prefrontal cortex E. Lateral temporal cortex

D Neurosciences

Low cerebrospinal fluid levels of which of the following neurotransmitter metabolites has been most consistently associated with suicidality? A. Homovanillic acid (HVA) B. 3-0-methy1-Dopa (3-OMD) C. Gamma-hydroxybutyrate (GHB) D. 5-hydroxyindole acetic acid (5-HIAA) E. 3-methoxy-4-hydroxyphenylglycoI (MHPG)

D Neurosciences

The addition of methyl groups to nucleotides in the promoter region of a gene has what effect on transcription? A. Upregulates microRNAs B. Forms double-stranded RNA C. Opens chromatin to allow greater access to DNA D. Prevents transcription factors from binding to DNA E. Phosphorylates histone proteins associated with DNA

D Neurosciences

The mesolimbic dopamine pathway includes which of the following structures? A. Hypothalamus B. Prefrontal cortex C. Substantia nigra D. Ventral striatum E. Periaqueductal gray matter

D Neurosciences

Which of the following best explains phenotypic differences between identical twins caused by environmental effects on genetic expression? A. RNA editing B. Somatic mutation C. RNA interference D. Epigenetic changes E. Post-translational modifications

D Neurosciences

Which of the following developmental syndromes results from a deletion that spans a number of genes? A. Fragile X B. Klinefelter C. Turner D. Williams E. Prader-Wi11i

D Neurosciences

Which of the following is a common form of sequence variation in the genome? A. Large deletions B. Microsatellite markers C. Simple tandem repeats D. Single nucleotide polymorphisms E. Simple sequence length polymorphisms

D Neurosciences

Which of the following neurohormones has a significant role in social bonding? A. Prolactin B. Corticotropin C. Somatostatin D. Oxytocin E. Melatonin

D Neurosciences

Which of the following neurotransmitters is most likely responsible for the rewarding nature of drugs of abuse? A. GABA B. Serotonin C. Glutamate D. Dopamine E. Norepinephrine

D Neurosciences

88. The role of glycine at the NMDA receptor can best be described as a(n): A. antagonist. B. partial agonist. C. inverse agonist. D. stearic modulator. E. obligate co-agonist.

E Neurosciences

A 5-year-old child presents with hyperuricemia, mental retardation, choreoathetosis and self-destructive biting of the lips and fingers. Which chromosome has been associated with this disorder? A. 4 B. 9 C. 15 D. 22 E. x

E Neurosciences

Decreased levels of which of the following neurotransmitters are most associated with depressed mood, poor sleep, and poor impulse control? A. Acetylcholine B. Dopamine C. Histamine D. Norepinephrine E. Serotonin

E Neurosciences

The presence of which of the following antibodies has been most consistently shown to substantially increase the risk of neuropsychiatric symptoms in lupus? A. Anti-Ro B. Anti-Hu C. Anti-Jo I D. Anti-thyroid E. Anti-phospholipid

E Neurosciences

Which of the following is a sign of an upper motor neuron injury? A. Diplopia B. Muscle atrophy C. Absence of Babinski signs D. Paresis with muscle flaccidity E. Hyperactive deep tendon reflexes

E Neurosciences

Potentially threatening objects produce a startle response before a person becomes consciously aware of them. This process is due to direct connections of the thalamus with which of the following? A. Amygdala B. Hippocampus C. Primary visual cortex D. Medial prefrontal cortex E. Autonomic nervous system

A Neurosciences

Small insertions, deletions, or duplications within the DNA molecule are referred to as: A. point mutations. B. nonsense mutations. C. chromatin remodeling. D. copy number variations. E. single nucleotide polymorphisms.

A Neurosciences

Which of the following conditions is most likely to be associated with the finding of upbeating nystagmus on primary gaze on neurologic exam? A. Medullary lesion B. Pontine hemorrhage or infarct C. Occult lung or breast carcinoma D. Craniopharyngioma in the optic chiasm region E. Congenital anomaly of the anterior third ventricle

A Neurosciences

49. The prevalence of AIDS in the US population increased after the discovery of zidovudine (AZT) and other effective treatments. Which of the following is the best explanation for this increase? A. More patients came forth with the diagnosis after treatment was discovered B. More patients contracted the illness, as there was less fear involved C. More patients with the illness were living longer D. More patients contracted the illness from birth

C

7. In order to make a DSM 5 diagnosis of attention-deficit hyperactivity disorder (ADHD), symptoms must be present prior to what age? A. 5 years B. 7 years C. 12 years D. 17 years E. 21 years

C

85. Adults with a history of childhood abuse are more likely than normal controls to have reduced size of which of the following structures on magnetic resonance imaging (MRI) scan of the brain? A. Caudate B. Amygdala C. Hippocampus D. Prefrontal E. Cerebellar vermis

C

The presence of which of the following would help in determining if a tonic-clonic seizure-like episode was non-epileptic rather than an epileptic seizure? A. Self-injury B. Stereotyped episodes C. Lack of elevation of prolactin D. Rhythmic movements of arms E. Presence of postictal confusion

C Clinical Neurology

A 70-year-old patient develops confusion, lethargy, and a generalized tonic-clonic seizure. Laboratory testing reveals a serum sodium of 95 mEq/l. Which of the following is the most likely complication of excessively rapid correction of this metabolic problem? A. Basilar artery stroke B. Dialysis encephalopathy C. Central pontine myelinolysis D. Portal-systemic encephalopathy E. Non-convulsive status epilepticus

C Clinical neurology

A patient with Parkinson disease develops a change in behavior characterized by frequent, compulsive gambling. Which of the following medications is most likely contributing to this change in the patient's behavior? A. Levodopa B. Selegiline C. Pramipexole D. Benztropine E. Entacapone

C Clinical neurology

Which of the following is the most effective treatment of focal hand dystonia associated with writing ("writer's cramp")? A. Carbamazepine B. Valproic acid C. Botulinum toxin D. Glatiramer acetate E. Levodopa/carbidopa

C Clinical neurology

An individualized education program (IEP) offers a wider range of options than a 504 plan for schools to meet children's educational needs because an IEP: A. has a lower standard for eligibility. B. allows accommodations for psychiatric illness. C. allows modification to the academic requirements for the student. D. allows schools to discipline any disruptive behavior as they see fit. E. allows schools to evaluate family members of the student if necessary.

C Clinical psychiatry Consultation/collaborative integrated care

32. A patient reports hearing the voice of some one who is not present, then stops moving, stares blankly, repetitively picks at clothing, and does not respond to questions or commands for several minutes. The confusion resolves after 15 minutes, but the patient has no recollections of these events. This likely represents which of the following types of seizure? A. Absence B. Myoclonic C. Simple partial D. Tonic-clonic E. Complex partial.

E

52. According to Winnicott, a caregiver who is sufficiently attuned and responsive to a child's needs to allow the child to become aware of the child's own drives is referred to as alan: A. true self. B. self object. C. auxiliary ego. D. transitional object. E. good-eilough parent.

E

58. Which of the following is the most common psychiatric complication of corticosteroid therapy? A. Dementia B. Delirium C. Psychosis D. Anxiety E. Mood disorder

E

59. Which of the following is the most appropriate dose of nicotine patches to give a patient who smokes 40 cigarettes per day? A. 7 mg/d B. 14 mg/d C. 21 mg/d D. 28 mg/d E. 42mg/d

E

6. In normal aging, which of the following cognitive abilities is the last to show decline? A. Perceptual speed B. Inductive reasoning C. Numerical ability D. Spatial orientation E. Word knowledge

E

9. A patient is brought to the emergency department in a state of psychosis caused by the use of inhalants. The patient is paranoid and anxious but cooperative. Which of the following is the best initial treatment of this acute condition? - A. Benzodiazepines B. Antipsychotics C. Inhaled oxygen D. Gastric lavage E. Watchful waiting

E

146. A 16-year-old high school student was placed on academic probation due to poor grades ool abs nces. In helping the family to cope effectively with their child's problems, a structural family theraJ.?itwould prioritize which of the following? A. Helping the parents to define expectations for their child that they all could support B. Ensuring that neither parent was scapegoating the child through projective identification C. Determining the goodness of fit between the school and the child's cognitive and emotional aptitudes D. Helping the family to become more egalitarian so the child would have a full voice in its deliberations

A

57. Initiatives by The World Health Organization to improve mental health services across the lifespan in low and middle-income countries have prioritized which of the following? A. Training non-specialist providers to identify and treat mental illnesses B. Sending physicians from these countries to the US for psychiatry training C. Developing psychiatric hospitals and treatment centers in these countries D. Developing training programs in these countries to produce more psychiatri ' E. Imbedding psychiatrists in schools and other government regulated organizations

A

86. The physiologic changes during REM sleep include increased eye movements, decreased muscle tone and which of the following? A. Increased heart rate B. Increased muscle activity C. Decreased blood pressure D. Decreased respiratio"ns E. Decreased cerebral blood flow

A

A 25-year-old patient is evaluated for daytime sleepiness. The patient reports falling asleep at night without difficulty and waking up at 7 a.m. every morning. The patient's spouse indicates that the patient does not snore. During the day the patient needs to take several brief naps which come on suddenly, and has fallen asleep while driving on one occasion. Sleep study reveals decreased REM latency. Which of the following would be the most effective treatment? A. Modafinil B. Zolpidem C. Oral appliance D. Avoid caffeine E. Continuous positive airway pressure

A Clinical neurology

A magnetic resonance imaging (MRI) scan of the head reveals an infarct in the distribution of the left anterior cerebral artery, Which of the following is the patient most likely to exhibit? A. Weakness of the contralateral foot and leg B. Incoordination of the ipsilateral arm and leg C. Visual field deficit consisting of homonymous hemianopia D. Ipsilateral third nerve palsy and contralateral hemiplegia E. Loss of pinprick sensation in the ipsilateral face and contralateral arm and leg

A Clinical neurology

A patient in the movement disorder clinic is evaluated for complaints of involuntary movements in the upper extremities which are described as sudden jerks, sometimes preceded by an urge to move. The movements are followed by a sense of relief. These movements are best characterized as: A. tics. B tremors. C. myoclonus. D. stereotypies. E. compulsions.

A Clinical neurology

Which of the following tests assesses for ideomotor apraxia in a patient with intact language comprehension? A. Asking the patient to pretend to light a match B. Giving the patient a pen and observing how he uses it C. Asking the patient to copy a picture of intersecting pentagons D. Having the patient tap his thumb and index finger together quickly E. Asking the patient to describe the steps needed to brush his teeth

A Clinical neurology

Federal courts have ruled that which of the following is a required component of psychiatric care in correctional institutions? A. Suicide prevention program B. Substance abuse counseling C. Participation by forensic psychiatrists D. Routine use of restraints and seclusion

A Clinical psychiatry Administration & systems

The operant procedure in which a desirable behavior pattern is learned by the successive reinforcement of approximations to that behavior is called: A. shaping. B. modeling. C. habituation. D. conditioning. E. sensitization

A Clinical psychiatry Behavioral & social sciences

Maternal smoking is known to be associated with which of the following effects on fetal physical development? A. Decreased birth weight B. Increased head circumference C. Accelerated lung development D. Decreased incidence of cleft palate E. Increased incidence of cardiac malformations

A Clinical psychiatry Development & maturation

The "false belief' test of young children uses two dolls in an interaction whereby one doll hides a marble in front of a second doll, and then moves the marble while the second doll leaves the scene. The second doll re-enters the scene and the child subject is then asked where this doll believes the marble to be. This test is used to assess which of the following? A. Theory of mind B. Imaginary play C. Conventional morality D. Verbal communication E. Nonverbal communication

A Clinical psychiatry Development & maturation

Which of the following is the most sensitive and specific laboratory test for identifying heavy drinking and monitoring treatment? A. Carbohydrate deficient transferrin (CDT) B. Mean corpuscular volume (MCV) C. Alkaline phosphatase (ALP) D. Alanine aminotransferase (ALT) E. Aspartate aminotransferase (AST)

A Clinical psychiatry Diagnostic procedures

A patient presents to the psychiatrist requesting an evaluation for dementia. He reports that his father had vascular dementia, and he wants to prevent a similar fate for himself. His past medical history is significant for diabetes, hypertension, and hyperlipidemia. On evaluation, there is no evidence of significant memory impairment or dementia. The psychiatrist recommends that the patient follow up with his primary care physician to optimize management of blood pressure, diabetes, and lipids. This recommendation is an example of which of the following? A. Primary prevention B. Secondary prevention C. Tertiary prevention D. Universal intervention E. Acute intervention

A Clinical psychiatry Epidemiology

In the Clinical Antipsychotic Trials of Intervention Effectiveness (CATIE) study, 60% of patients with schizophrenia reported some substance abuse (not including nicotine). Which of the following substances was most commonly used? A. Alcohol B. Cocaine C. Marijuana D. Stimulants E. Methamphetamines

A Clinical psychiatry Epidemiology

Which of the following is a risk factor for child abuse? A. Financial stress B. Small family size C. Two-parent household D. Child older than 13 years E. College educated parents

A Clinical psychiatry Epidemiology

An outpatient clinic requires patients to show two pieces of identification before every appointment. This policy helps ensure that all care provided is: A. safe. B. timely. C. efficient. D. effective. E. patient-centered

A Clinical psychiatry Issues in practice

A patient who recently delivered a full term newborn is brought to the emergency department by her husband. The patient is agitated, labile, and demanding to leave the hospital because she says that her baby is being poisoned. This condition is considered a psychiatric emergency because of its association with: A. infanticide. B. bipolar disorder. C. substance abuse. D. psychotic disorder. E. high recurrence rate.

A Clinical psychiatry Psychopathology & associated conditions

Which of the following is a paramnesia? A. Déjå vu B. Melancholia C. Folie å deux D. La belle indifference E. Pseudologia phantastica

A Clinical psychiatry Psychopathology & associated conditions

In addition to atypical antipsychotics, which of the following augmenting agents is most effective in depressed patients with insufficient response to antidepressant monotherapy? A. Lithium B. Valproate C. Lamotrigine D. Levothyroxine E. Methamphetamine

A Clinical psychiatry Treatment

The dialectical behavioral therapist primarily focuses on which of the following? A. Utilizing contingencies and skills to replace the dysfunctional behavior B. Creating a holding environment to analyze the cause of dysfunctional behavior C. Encouraging expression of intense affect and encouraging disruptive behavior D. Facilitating expression of intense feelings, providing support for these feelings, and using gradual tasks to substitute for agitation

A Clinical psychiatry Treatment

Abnormal emotional expression such as pathological laughter or crying is most likely caused by lesions affecting cortical-subcortical circuits linking the frontal cortex, pons and which of the following? A. Cerebellum B. Dentate gyrus C. Raphe nucleus D. Intralaminar thalamic nuclei E. Nucleus of the trigeminal nerve

A Neurosciences

Allelic exclusion and X-chromosome inactivation can be best explained by which of the following mechanisms? A. Epigenetic B. Post-translational C. Somatic mutation D. Post-transcriptional E. Environmental influences

A Neurosciences

Which of the following developmental disorders is a trinucleotide repeat disorder? A. Friedreich ataxia B. Klinefelter syndrome C. Turner syndrome D. Williams syndrome E. Angelman syndrome

A Neurosciences

Which of the following is the gene mapping strategy that involves identification of genetic markers that co-segregate with disease phenotype? A. Pedigree analysis B. Affected sib-pair analysis C. Family trios genome wide association D. Case control genome wide association

A Neurosciences

137. A 69-year-old patient presents with §Udden onf headaVQ..ryiti..ng. The patient is unable to sit or stand, has mild left facial weakriesS, wea:Kiiess oLthe left lateral rectus and reduced corneal reflex on the left eye. The rest of the examination is uiireiilafkable. A computed tomography (CT) scan of the head is shown below. Which of the following is the most likely diagnosis? A. Acute hydrocephalus B. Hypertensive hemorrhage C. Subarachnoid hemorrhage D. Posterior circulation infarct

B

142. A 20 year-old patient is evaluated for bouts of mild depression and weight loss of 10 pounds. The patient admits strenuously exercising many hours per day sometimes missing work. She reports, "There are days I lose it adn I et tons of my favorite sweets, I can eat a big cake in a few minutes. So, I exercise to stay in good shape, to look good, which is very important for me." The patient reports that this behavior has been going on for about a year. Physical examination is normal and height, weight, vital signs and laboratory tests are within normal limits. Which of the following disorders is the most probable diagnosis? · A. Binge eating B. Bulimia.nervosa c. Anorexia nervosa D. Avoidant restrictive food intake disorder

B

A 60-year-old man with a history of liver disease presents with confusion. On examination the patient demonstrates asterixis of the hands and is found to have an elevated serum ammonia level. Electroencephalograph (EEG) is likely to demonstrate which of the following? A. Normal alpha rhythm B. Bilateral triphasic waves C. Focal slowing in the temporal lobe D. Continuous spike wave discharges E. Periodic lateralized epileptiform discharges

B Clinical neurology

Inability to name objects held in the left hand, but not in the right hand, indicates a lesion in which of the following? A Left frontal lobe B CoIpus callosum C Left parietal lobe D Right parietal lobe E Anterior commissure

B Clinical neurology

Which of the following is the causative agent of progressive multifocal leukoencephalopathy? A Prions B. JC virus C. Cytomegalovirus D. Herpes simplex virus type I E. Human herpes virus type 6

B Clinical neurology

Use of mental health courts for qualified defendants has been shown to increase which of the following? A. Remorse for criminal acts B. Engagement in community treatment C. The number of inmates treated in prison D. Long-term abstinence from illicit substances E. Overall cost to the justice and mental health system

B Clinical psychiatry Administration & systems

A patient presents to the psychiatrist for evaluation of mental and physical exhaustion, lightheadedness, difficulty concentrating, headaches, and gastrointestinal distress. The symptoms have been present for about 6 months and first began when the patient immigrated to the United States from China, Since arriving in the U.S., the patient has struggled to find work and has been living with relatives. In assessing this patient's symptoms, which of the following questions is most appropriate? A. "Do you think you are depressed?" B. "What do you think is causing your symptoms?" C. "Do you know anyone else with these symptoms?" D. "On a scale of 1-10, how severe is your exhaustion?" E. "Are you willing to take medications to help you feel better?"

B Clinical psychiatry Behavioral & social sciences

According to Winnicott, a "good enough" parent is attuned to the infant's mental state most of the time. The parent communicates this understanding to the infant via which of the following processes? A. Holding B. Mirroring C. Projecting D. Interpreting E. Internalizing

B Clinical psychiatry Behavioral & social sciences

A consultation-liaison psychiatrist evaluates a 40-year-old patient who has had an uncomplicated hospital course following surgery under general anesthesia. On post-op day four the patient develops new-onset hallucinations. Which of the following is the most likely cause? A. Schizophrenia B. Delirium tremens C. Acute stress disorder D. Brief psychotic disorder E. Major depression with psychotic features

B Clinical psychiatry Consultation/collaborative innovative care

At what age does a typically developing child usually first speak in complete sentences? A. 2 years B. 3 years C. 4 years D. 5 years E. 6 years

B Clinical psychiatry Development & Maturation

Third- and fourth-grade students from several different classrooms are sharing the school playground. Which of the following is likely to be the most dominant observable social pattern? A. Boy-girl pairing for role-playing games B. Same-sex groupings regardless of activity C. Initiation of boy-girl, mixed-grade games based upon shared interests D. Same-grade cohorts avoiding social contact with students from other grades E. Emergence of a charismatic leader who directs the play for the entire group

B Clinical psychiatry Development & Maturation

Which of the following metabolic changes is characteristic of normal aging? A. Insulin levels decline B. Creatinine clearance declines C. Corticotropin levels decrease D. Parathyroid hormone levels decline

B Clinical psychiatry Development & Maturation

A sense of idealism and invulnerability that may lead to risky behavior is common in which phase of life? A. Childhood B. Adolescence C. Young adulthood D. Middle adulthood E. Late adulthood

B Clinical psychiatry Development & maturation

A patient is instructed to name the colors in which the series of words are printed, rather than reading the words themselves. This is called the: A Tower Test. B Stroop Test. C Boston Naming Test. D Test of Auditory-Perceptual Skills. E Wisconsin Card Sorting Test.

B Clinical psychiatry Diagnostic procedures

After discontinuing clozapine due to WBC count less than 2,000/mm3 or absolute neutrophil count less than 1,000/mm3, for how long must a patient continue to have a weekly CBC performed once WBC count returns to more than 3,500/mm3 ? A. 1 week B. 4 weeks C. 2 months D. 6 months E. 12 months

B Clinical psychiatry Diagnostic procedures

Which of the following types of electroencephalogram (EEG) monitoring is most likely to aid in differentiating psychogenic from true blindness? A. Sleep deprivation B. Evoked potentials C. 24-hour monitoring D. Polysomnography E. Nasopharyngeal leads

B Clinical psychiatry Diagnostic procedures

Most patients presenting for detoxification from alcohol are treated on an outpatient basis, Which of the following risk factors would lead most clinicians to recommend inpatient treatment for a new patient? A. Age under 50 years B. History of a prior withdrawal seizure C. First-degree relative with delirium tremens D. Hepatic enzymes elevated twice the upper limit of normal E. History of multiple arrests for driving under the influence

B Clinical psychiatry Epidemiology

Psychological stress increases cortisol synthesis and release, and prolonged, increased cortisol levels have been associated with adverse psychiatric outcomes. Stress also affects the release of other hormones. An increase of which of the following hormones is correlated with decreased posttraumatic stress disorder severity, most likely due to the hormone's anti-glucoconicoid properties? A. Prolactin B. Dehydroepiandrosterone (DHEA) C. Thyroid-stimulating hormone (TSH) D. Corticotropin-releasing hormone (CRH) E. Adrenocorticotropic hormone (ACTH)

B Clinical psychiatry Epidemiology

According to the American Psychiatric Association guidelines, sexual relationships with patients are: A. permitted with former patients only. B. not allowed with current or former patients. C. permitted with parents of child patients, but not with patients themselves. D. permitted when the patient fully understands the implications of the relationship and gives consent. E. not allowed until the psychiatrist terminates the relationship and refers the patient to another provider.

B Clinical psychiatry Issues in Practice

When an incarcerated adult patient is being treated in a correctional setting, a limit to confidentiality that does not exist in other settings is information about: A. communicable diseases such as HIV. B. imminent plans to escape or create disorder. C. threats made against the patient by other inmates. D. illegal activity, but only if it is related to the crime for which the patient is incarcerated.

B Clinical psychiatry Issues in Practice

Eighty percent of the patients in a substance use program are engaged in treatment. Which of the following types of quality measures does this represent? A. Structure B. Process C. Outcome D. Screening E. Satisfaction

B Clinical psychiatry Issues in practice

Which of the following is the infectious neuropsychiatric disorder characterized by psychosis and abnormal gait, that was once treated with malaria-induced fever and accounted for up to 20% of patients in mental asylums until the 1940s? A. Polio B. Syphilis C. Lyme disease D. Herpes simplex virus E. Post-streptococcal infection

B Clinical psychiatry Issues in practice

Which of the following strategies has been most often linked to better outcomes in the treatment of depression in primary care settings? A. Patient education B. The use of care managers C. Continuing medical education D. Published treatment guidelines E. Depression screening measures

B Clinical psychiatry Issues in practice

A 50-year-old patient presents to the emergency department with acute onset of auditory and visual hallucinations. The hallucinations abruptly began several hours prior to presentation. The patient also complains of recently feeling "moody" and "on edge." The patient is alert and fully oriented. Physical examination is notable for tachycardia and diaphoresis. Which of the following disorders does the patient most likely have? A. Opioid use disorder B. Alcohol use disorder C. Brief psychotic disorder D. Bipolar I disorder with psychotic features E. Major depression with psychotic features

B Clinical psychiatry Psychopathology & associated conditions

A patient with a depressive disorder presents with increased appetite and weight gain, hypersomnia, and a heavy feeling in the arms and legs. Which of the following is the most likely mood description? A Irritable B Mood reactivity C Keyed up and tense D Worse in the morning E Elevated and expansive

B Clinical psychiatry Psychopathology & associated conditions

Which of the following is a core feature of factitious disorder? A. Symptoms must be present for at least one month B. Intentional falsification of medical or psychological symptoms C. Association of an obvious external reward with the illness behavior D. Symptoms are incompatible with known medical or neurological illness E. Persistence of belief that one is ill when presented with evidence to the contrary

B Clinical psychiatry Psychopathology & associated conditions

A 58-year-old patient with major depressive disorder is undergoing a course of ECT treatment. After the third ECT treatment, the patient develops a dense retrograde amnesia. This side effect can be ameliorated by which of the following? A. Switching from unilateral to bilateral ECT B. Increasing the interval between ECT treatments C. Decreasing the interval of the ECT-induced seizures D. Concurrent administration oftacrine during the course of ECT E. Concurrent administration of methylphenidate during the course of ECT

B Clinical psychiatry Treatment

A patient with Alzheimer disease has been treated with donepezil for the past few years. The dementia has progressed, and the patient now presents with significant agitation, lability, and appetite disruption. A medical workup is unrevealing. Which of the following would be the most appropriate medication to add to donepezil? A. Citicoline B. Memantine C. Galantamine D. Rivastigmine E. Physostigmine

B Clinical psychiatry Treatment

Biofeedback is a treatment modality used across multiple disorders to help patients achieve greater control over specific physiological functions via which of the following mechanisms? A. Classical conditioning B. Operant conditioning C. Covert sensitization D. Stimulus generalization E. Observational learning

B Clinical psychiatry Treatment

Dialectical behavior therapy is unique among commonly employed psychotherapies in encouraging judicious use by patients of which of the following? A. Keeping a diary B. Telephone consultation C. Brief hospital admissions D. Psychotropic medication E. Consultation with other therapists

B Clinical psychiatry Treatment

Interpersonal psychotherapy of adults with depression begins with assessment of the patient's adjustment to stressful life events, role transitions and disputes, and: A. ability to learn from interpretation. B. quality of social connections. C. early childhood experiences. D, unconscious conflicts. E. character traits.

B Clinical psychiatry Treatment

Which of the following characteristics of self-help groups distinguishes them from traditional psychotherapy groups? A. Self-help groups focus on modifying personality. B. Goals and objectives do not address intragroup dynamics. C. The leader prohibits extragroup socializing and social contacts. D. Professional involvement of the leader is central to establishing goals. E. Instillation of hope is absent because the specific problem is too far advanced.

B Clinical psychiatry Treatment

Which of the following second generation antipsychotic medications carries package labeling by the FDA highlighting the risk of QTc prolongation? A Quetiapine B Paliperidone C Aripiprazole D Olanzapine E Clozapine

B Clinical psychiatry Treatment

Exposure to light helps entrain the circadian system by activating nonvisual retinal photoreceptors that project to which of the following brain structures? A. Pineal gland B. Suprachiasmatic nucleus C. Tuberomamillary nucleus D. Reticular activating system E. Ventrolateral preoptic nucleus

B Neurosciences

Genetic mapping methods aim to identify disease-associated variants based on which of the following? A. Epigenetics B. Genetic linkage C. Genetic mutation D. Copy number variants E. Phenotypic expression

B Neurosciences

Narcolepsy is believed to be caused by deficiency of which of the following substances? A. Melatonin B. Hypocretin C. Acetylcholine D. Norepinephrine E. Melanin-concentrating hormone

B Neurosciences

There is a strong association between HLA B1502 allele and Steven-Johnson syndrome in individuals of Asian descent who are treated with which of the following agents? A. Valproic acid B. Carbamazepine C. Lamotrigine D. Topiramate E. Phenytoin

B Neurosciences

An 87-year-old patient who is a college graduate with longstanding cardiovascular disease and dyslipidemia is brought to the psychiatrist by an adult daughter who lives with the patient and has become concerned about the patient's forgetfulness and confusion. The patient receives a score of 21/30 on the Folstein Mini-Mental State Examination (MMSE). This score is consistent with: A. normal cognitive decline. B. mild cognitive impairment. C. moderate cognitive impairment. D. severe cognitive impairment. E. probable sensory impairment.

B (I disagree with this) Clinical psychiatry Diagnostic procedures

61. At what age is normal genital self-stimulation particularly pronounced in children? A. 6 months B. 18 months C. 3 yearsf D. 5 years E. 7 years

BC

106. A young girl remarks, "I am going to stay a girl no matter what, even if I wear blue or have my hair cut short)" Which of the following capacities has this child achieved? A. Gender role B. Sexual orientation C. Gender constancy D. Gender identity E. Gender labeling

C

114. According to the theory of self-psychology, a major cause of mental illness is: A. an overly anxious mother. B. an inability to gratify unconscious drives. C. absence of age-specific mirroring responses. D. excessive activation of maladaptive schemas. E. failure to integrate good and bad images of the world.

C

117. In contrast to attachment, "bonding" is associated with: A. a critical period. B. an infant's exploration of a new environment. C. the, nature of a parent's relationship to an infant. D. an infant's response to the parent's departure and return. E. specific patterns of an infant's responsiveness to a caretaker.

C

138. An 82-year-old patient has a several-year history of falls, ophthalmoplegia, parkinsonism and progressive dementia. Which of the following is the most likely finding at autopsy? A. Beta amyloid positive plaques B. Tau positive achromatic neurons C. Tau positive neurofibrillary tangles D. Alpha synulclein positive intraneuronal inclusions

C

47. Patients taking lithium are more likely to show elevated levels of which of the following serum electrolytes independent of renal function? A. Sodium B. Potassium C. Calcium D. Magnesium E. Chloride

C

The 5-HT3 receptor is best described as a(n): A. intra-cellular receptor. B. mitochondrial receptor C. ligand gated ion channel. D. voltage gated ion channel. E. G-protein coupled receptor.

C Neurosciences

A 62-year-old patient with poor nutrition complains of difficulty walking. Examination is remarkable for hyperreflexia with marked reduction in vibration and position sense at the toes. A diagnosis of cobalamin (vitamin 1312) deficiency is suspected, but the results of cobalamin (vitamin B12) testing reveal a borderline low value. Which of the following lab tests would confirm the suspected diagnosis? A. Folate B. Glutathione C. Methylmalonic acid D. Pyridoxine (vitamin B6) E. Erythrocyte sedimentation rate

C Clinical neurology

An 80-year-old patient is evaluated for visual hallucinations and worsening gait. The patient's spouse reports that the patient has had episodic confusion and disturbed sleep in which the patient shouts out and appears to be fighting someone. Previous trials of levodopa/carbidopa have resulted in temporary improvement in motor function. Examination is remarkable for bilateral rigidity, masked facies and gait instability. Which of the following is the most likely diagnosis? A. Alzheimer disease B. Multisystem atrophy C. Dementia with Lewy bodies D. Idiopathic Parkinson disease E. Corticobasal ganglionic degeneration

C Clinical neurology

Which of the following types of medication can precipitate REM sleep behavior disorder? A. Stimulant B. Benzodiazepine C. Selective serotonin reuptake inhibitor D. Typical (first generation) antipsychotic E. Atypical (second generation) antipsychotic

C Clinical neurology

Which of the following is the strongest predictor of engagement in leadership roles by patients in mental health consumer-run organizations? A. Race or ethnicity of patient B. Availability of peer counseling C. Perceived sense of community D. Perceived organizational empowerment E. Number of self-help group meetings offered

C Clinical psychiatry Administration & systems

Proponents of which of the following theories agree that social norms, expectations, meaning and perceptions are keys to understanding individual behavior in institutions and institutional effectiveness? A. Open systems B. Human relations C. Organizational culture D. Structural and sociotechnical E. Organizational power and conflict

C Clinical psychiatry Administrations & systems

A bookkeeper who is embezzling money justifies this behavior by noting that the company is still making a profit. This scenario exemplifies which of the following defense mechanisms? A. Idealization B. Sublimation C. Rationalization D. Intellectualization E. Projective identification

C Clinical psychiatry Behavioral & social sciences

When treating psychiatrists are asked by the judicial system to testify on behalf of their patients, ethical considerations, the doctor-patient relationship, and legal guidelines indicate that they can comment on which of the following topics? A Damages B Impairment ratings C Clinical presentation D Causation of injury E Extent of disability

C Clinical psychiatry Consultation/collaborative integrated care

A 38-year-old single woman presents to the psychiatrist reporting difficulty dealing with the prospect of not having children. She has always wanted to be a mother, and given her age and lack of current romantic involvement, she fears that motherhood may not be possible. She becomes a little tearful as she discusses this, and wonders whether she should consider adoption as a single mother. Which of the following is the most likely diagnosis? A. Relational problem B. Adjustment disorder C. Phase of life problem D. Generalized anxiety disorder E. Unspecified depressive disorder

C Clinical psychiatry Development & Maturation

The adult child of a 75-year-old parent is concerned that the parent is no longer safe to drive, after getting into several recent car accidents. The parent is in good physical health, has no chronic diseases, and is on no medications. Which of the following is most likely the cause of the motor vehicle accidents? A. Hearing loss B. Hand tremors C. Slower visual processing D. Impaired word processing E. Decreased muscle strength

C Clinical psychiatry Development & Maturation

Increased risk-taking behavior is most likely to occur as a result of a lesion in which of the following brain regions? A. Precuneus B. Cerebellar vermis C. Orbital frontal cortex D. Medial temporal cortex E. Parahippocampal gyrus

C Neurosciences

Which of the following interview techniques is being used when asking the patient, "What brought you to the hospital today?" A Confrontation B Attentive listening C Open-ended questioning D Maintenance of neutrality E Closed-ended questioning

C Clinical psychiatry Diagnostic procedures

A media campaign is launched to reduce the incidence of suicides by military veterans. The campaign includes television, radio, and print advertisements that provide information about warning signs, directions on how to get help, and evidence about the effectiveness of treatment. This is an example of which of the following public health strategies? A. Secondary prevention B. Teniary prevention C. Universal intervention D. Selective intervention E. Indicated intervention

C Clinical psychiatry Epidemiology

A patient with moderately severe dementia participates in a randomized controlled trial of a new cognitive enhancer. Proxy consent was obtained from the patient's spouse, After 3 weeks, the patient tells the psychiatrist he no longer wishes to participate in the study because of nausea he attributes to the drug but the spouse wants the patient to continue taking it. Which of the following should the psychiatrist do? A. Continue the patient in the study as it is in his best interest B. Continue the patient in the study as the spouse has provided proxy consent C. Withdraw the patient from the study because not doing so breaches his autonomy D. Withdraw the patient from the study because not doing so may cause significant harm

C Clinical psychiatry Issues in Practice

Which of the following would be most useful for enhancing the doctor-patient relationship and improving emotional communication when using telemedicine to interview a patient? A. Increasing the volume of the audio to near maximum volume B. Recording the interchange so that the patient can review it later C. Positioning the webcam close to the image of the patient on the screen D. Using a wide zoom so that the psychiatrist can see the patient's entire body E. Using shorter sentences with more frequent pauses than would be done in person

C Clinical psychiatry Issues in practice

A 17-year-old patient is brought by family to the emergency department due to severe symptoms of restlessness, nervousness, facial flushing, muscle twitching, and tachycardia. Family became most concerned when the patient began to have rambling flow of thought and speech, and was becoming agitated. The patient has most likely been drinking a large quantity of: A ginger ale. B. sarsaparilla. C. energy drinks. D. ginseng soda. E. pomegranate juice.

C Clinical psychiatry Psychopathology & associated conditions

A male physician is repeatedly accused by adult female patients of sitting opposite them on his stool during an eye examination, opposing knees alternating, in such a manner that one of the patients' knees touches the physician's crotch, If true, this is an example of: A. fetishism. B. partialism. C. frotteurism. D. sexual sadism. E. sexual masochism.

C Clinical psychiatry Psychopathology & associated conditions

A patient consults the psychiatrist for nervousness felt in many public situations. The patient, who has a very mild stutter, fears being viewed by others as stupid and inept. The patient reports being in a very stable marriage with mutually satisfying intimacy but was recently passed over for a promotion at work due to "not being a team player. Most of the patient's meals are eaten at home, as dining in restaurants and other public places elicits discomfort. Which of the following is most likely? A. Agoraphobia B. Normal shyness C. Social anxiety disorder D. Avoidant personality disorder E. Anxiety related to a medical condition

C Clinical psychiatry Psychopathology & associated conditions

A patient presents for an evaluation of "memory problems. The patient reports frequent episodes of ending up somewhere without any memory of getting there. The patient also occasionally forgets how to perform job functions, and has gaps in memories of high school. In addition, the patient endorses the occasional experience of observing the self from outside the body, and notes periods of being suddenly overcome by unexpected emotions. Recently the patient began hearing a child's voice. Which of the following is the most likely diagnosis? A. Malingering B. Dissociative amnesia C. Dissociative identity disorder D. Borderline personality disorder E. Depersonalization/derealization disorder

C Clinical psychiatry Psychopathology & associated conditions

Hoarding disorder is most commonly accompanied by which of the following disorders? A. Bipolar B. Agoraphobia C. Major depressive D. Obsessive-compulsive E. Attention-deficit/hyperactivity

C Clinical psychiatry Psychopathology & associated conditions

When evaluating a patient in the emergency department, which of the following symptoms most strongly suggests a general medical condition? A Anergia B Irritability C Clouded sensorium D Psychomotor agitation E Auditory hallucinations

C Clinical psychiatry Psychopathology & associated conditions

Which of the following is a predisposing vulnerability factor in posttraumatic stress disorder (PTSD)? A. Male gender B. Schizotypal traits C. Recent excessive use of alcohol D. Perception of internal locus of control E. Obsessive-compulsive personality traits

C Clinical psychiatry Psychopathology & associated conditions

A married couple is seeking therapy to help with issues of mutual resentment. The wife has already been in individual therapy for several months and has a high regard for her therapist. She asks if her therapist can conduct the couples therapy as well. Which of the following is the therapist's best response? A. "I would be happy to. I really think your husband needs to hear what you been going through. " B. "No, it would be better for your husband's therapist to do the therapy, as he can represent your husband's point of view. " C. "A couples therapist needs to be impartial, and I already have a relationship with you. " D. "I'd enjoy that, but I can't fit you into my schedule right now; let me suggest a colleague.

C Clinical psychiatry Treatment

A patient with traumatic brain injury is trained to use an electronic tablet to keep track of appointments and other details of life, and works monthly with a therapist to help successfully integrate the use of it into everyday life. Which of the following interventions best describes this type of intervention? A. Milieu therapy B. Harm reduction C. Cognitive rehabilitation D. Motivational interviewing E. Supportive psychotherapy

C Clinical psychiatry Treatment

Co-prescription of which of the following analgesic medications is likely to cause a clinically significant rise in lithium level? A. Acetaminophen B Amitriptyline C. Celecoxib D. Duloxetine E. Oxycodone

C Clinical psychiatry Treatment

The purpose of adding oral naloxone to buprenorphine for opioid replacement therapy is because its opioid antagonism prevents which of the following? A. Euphoria from the buprenorphine B. Overdose if extra doses are ingested C. Intravenous abuse of the buprenorphine D. Euphoria if additional types of opioids are used E. Overdose if additional types of opioids are used

C Clinical psychiatry Treatment

Which of the following physiologic changes best explains why older adults, especially people over age 70, are at increased risk of developing extrapyramidal symptoms during antipsychotic medication treatment? Decreased: A. monoamine oxidase activity B. acetylcholinesterase activity C. number of available dopamine receptors D. renal clearance of antipsychotic medications E. volume of distribution of antipsychotic medications

C Clinical psychiatry Treatment

A patient with autism has had significant difficulties with disruptive behavior, loud outbursts, and functional difficulties over the past year. The patient has been actively engaged in outpatient treatment without significant progress despite intensive psychotherapy and psychopharmacologic treatment. There is no evidence of suicidality or aggression. Which of the following is the most appropriate treatment step for this patient? A. Boarding school B. Residential treatment C. Partial hospitalization program D. Inpatient admission for acute stabilization E. Continue outpatient treatment with closer follow-up

C Clinical psychiatry Treatment

Hyperprolactinemia with antipsychotic medication use increases risk of which of the following? A. Seizures B. Hypersexuality C. Osteoporosis D. Hallucinations E. Heart disease

C Neurosceinces

Clear evidence for adult neurogenesis has been established in which of the following areas of the brain? A. Amygdala B. Cerebellum C. Hippocampus D. Olfactory bulb E. Anterior cingulate cortex

C Neurosciences

Conical columns are vertical collections of neurons whose role is best understood as: A. signaling units for critical periods. B. regulatory units for long-term potentiation. C. functional units for information processing. D. redundant backups for subcortical processing. E. reserve units underlying experience-dependent plasticity.

C Neurosciences

Which of the following methods is used to confirm the diagnosis of velocardiofacial syndrome (22q.11 microdeletion)? A. Karyotyping B. Genetic linkage analysis C. Fluorescent in situ hybridization D. Trinucleotide expansion analysis E. Mitochondrial DNA deletion analysis

C Neurosciences

98. Which of the following abnormalities of speech or language is a patient with autism spectrum disorder most likely to demonstrate? A. Dyslexia B. Aphasia C. Impaired prosody D. Pressured speech E. Problematic enunciation

C lack of pitch , high and lows of voice more like monotone of voice

95. A patient is brought to the emergency department by family members who report that the patient has a history of depression and has recently been complaining of hearing voices. Two days ago, the patient stopped speaking and began maintaining unusual postures for hours at a time. During examination moving. Which of the following is the most appropriate pharmacologic intervention? A. Haloperidol B. Olanzapine C. Fluoxetine D. Lorazepam E. Methylphenidate

D

138. Abortive treatment of common migraine is best achieved with which of the following medications? A. Duloxetine B. Topiramate C. Desipramine D. Rizatriptan

D Clinical neurology

A 28-year-old woman is hospitalized with symptoms of paranoid delusions, auditory hallucinations, and agitation. When treated with haloperidol 5 mgs the patient is noted to become rigid and mute. Within days she develops choreiform movements, has a seizure and develops respiratory difficulty leading to an ICU admission. An examination of the patient, which includes an abdominal ultrasound, reveals an ovarian mass. A lumbar puncture is performed, Cerebrospinal fluid (CSF) is likely to show antibodies to which of the following? A. VGKC-complex B. AMPA receptor C. GABA B receptor D. NMDA receptor E. Glycine receptor

D Clinical neurology

A 63-year-old patient with a history of excess alcohol intake presents with confusion. The patient is unaware of surroundings and answers only some questions with brief responses. On examination the patient is unable to move eyes laterally on either side and has nystagmus when attempting lateral gaze. The patient is unable to stand without assistance. The patient's symptoms are most likely a result of deficiency of: A. iron. B. folate. C. glucose. D. thiamine. E. vitamin D.

D Clinical neurology

A 65-year-old patient is brought to the emergency department with memory problems and confusion over the past two days. The patient is unable to follow simple commands and disoriented on examination with no apparent focal motor deficits. A magnetic resonance imaging (MRI) scan of the brain demonstrates increased signal in the bilateral medial temporal and orbitofrontal regions and electroencephalogram (EEG) shows periodic discharges over the left temporal lobe. Based on the information provided which of the following would be the likely treatment for this condition? A. Ceftriaxone B. Mannitol C. Heparin D. Acyclovir E. Prednisone

D Clinical neurology

Studies of education campaigns aimed at reducing the stigma of mental illness have reached the unexpected finding that: A. being given a treatment for a mental illness, even if it is effective, increases stigma. B. the public is more accepting of mental illnesses that are considered to be unpredictable. C. stigma around mental illness is not related to the public perception of dangerousness. D. biological explanations for mental illness are not associated with more tolerant attitudes to the illness.

D Clinical psychiatry Behavioral & Social Sciences

A patient has the goal of cleaning out the garage, but becomes extremely anxious and overwhelmed by the thought of even starting to work on it. The patient's therapist suggests breaking the job into manageable pieces to make it seem less overwhelming. Which of the following cognitive-behavioral techniques is this describing? A. Activity scheduling B. Relaxation training C. Behavioral activation D. Graded task assignments E. Exposure and response prevention

D Clinical psychiatry Behavioral & social sciences

Salivation in response to the smell of dinner represents which element of classical conditioning? A. Habituation B. Conditioned stimulus C. Conditioned response D. Unconditioned response E. Unconditioned stimulus

D Clinical psychiatry Behavioral & social sciences

Which of the following is the most common fear expressed by adults leaving home to enter hospice? A. Pain B. After-life C. Losing control D. Separating from loved ones E. Becoming a burden on others

D Clinical psychiatry Development & Maturation

According to the 2013 Youth Risk Behavior Surveillance Survey which of the following is a recent trend in adolescent sexuality? A. Most adolescents experience sexual intercourse. B. Adolescents' first sexual experience is almost never with a steady relationship partner. C. The proportion of sexually experienced adolescents has increased since the mid-1980's. D. Early age at first sexual intercourse is associated with having more sexual partners. E. About half of never-married adolescents have had six or more partners since their sexual debut.

D Clinical psychiatry Development & maturation

The primary psychological task of school-age children is achieving which of the following? A. Identity B. Autonomy C. Attachment D. Competence E. Individuation

D Clinical psychiatry Development & maturation

Which of the following statements is accurate about sexuality and aging? A. Older women are more sexually active than older men. B. Past levels of sexual activity do not predict later levels of activity. C. Fewer than half of older adults express satisfaction with their sex lives. D. A majority of middle-aged and older individuals continue to be sexually active. E. Physical health is the most important factor determining sexual activity for both sexes.

D Clinical psychiatry Development & maturation

The psychiatrist evaluates a patient whose spouse describes the patient's personality change over the past year including shoplifting, lying, flirtatious behavior, craving foods with sugar, and hoarding canned goods. The patient has no past psychiatric history, and scores 20/30 on the Montreal Cognitive Assessment (MoCA). Which of the following would be the most useful in supporting a diagnosis? A. Electroencephalogram (EEG) B. Diffusion tensor imaging (DTI) C. Dopamine transporter (DaT) scan D. Single-photon emission computed tomography (SPECT)

D Clinical psychiatry Diagnostic procedures

102. Which of the following tools can be used when a person wishes to assign a surrogate to make health care decisions during future periods of mental incapacity? A. Guardianship B. Conservatorship C. Guardian ad litem D. Advance directive E. Representative payee

D Clinical psychiatry Issues in practice

36. In the early stages, Huntington disease is characterized by loss of neurons that produce which of the following neurotransmitters? A. Dopamine B. Acetylcholine C. Glutamate D. GABA E. Serotonin

D Neurosciences

A psychiatrist has recently graduated from residency training and is setting up a private practice. The psychiatrist is personally an avid user of social media and wants patients to be able to access the office through social media. The psychiatrist should: A. send "friend" requests to all new patients. B. not send "friend" requests to patients, but accept them from patients. C. not use social media for professional purposes as it violates patient privacy. D. set up a professional account completely separate from any personal accounts. E. not use social media professionally, but use email to communicate with patients and potential patients.

D Clinical psychiatry Issues in practice

Among individuals with anorexia nervosa, those with binging-purging type are more likely than those with restrictive type to present with comorbid: A. anxiety. B. depression. C. malnutrition. D. substance use.

D Clinical psychiatry Psychopathology & Associated Conditions

Withdrawal from which of the following substances can constitute a life-threatening medical emergency? A. Cocaine B. Opioids C. Phencyclidine (PCP) D. Sedative-hypnotics E. MDMA (ecstasy)

D Clinical psychiatry Psychopathology & Associated Conditions

Which of the following is the most clinically significant critique of the use of randomized controlled studies as the gold standard level of evidence? A. They are impractical, costly, and too difficult to perform in most situations. B. They tend to investigate hypotheses that can be more easily answered by other study designs. C. True randomization is a mathematical impossibility, thus group assignments are always biased. D. Their design is better suited to provide information about efficacy rather than effectiveness. E. They are more useful for investigating negative (i.e., adverse) effects than positive outcomes,

D Clinical psychiatry Research & scholarly literacy

The reliability of an assessment instrument, such as a diagnostic interview, refers to the ability of an instrument to: A measure what it intends to measure. B yield low false-positive rates of mistakenly identifying subjects. C. yield adequate information without being unduly time consuming. D. yield consistent results when used by different examiners or at different times. E. detect true cases of particular disorders, thus yielding low false-negative rates.

D Clinical psychiatry Research & scholarship literacy

Which of the following is the main advantage of parallel group, randomized controlled designs compared to crossover designs? A. Less expensive B. More time required C. Increased precision D. Fewer residual effects E. Fewer subjects needed

D Clinical psychiatry Research & scholarship literacy

A patient with bipolar disorder has been clinically stable on Lamotrigine 200 mg daily for the past 6 months. The patient contacts the psychiatrist to request a refill of Iamotrigine, explaining that, "I ran out five days ago but kept forgetting to call." The patient denies any mood symptoms or physical complaints. The most appropriate next step is to restart the lamotrigine at: A. full dose (200 mg daily). B. half the prior dose (100 mg daily). C. 25 mg daily and titrate to 200 mg daily over I week. D. 25 mg daily and follow standard initial titration schedule.

D Clinical psychiatry Treatment

Which of the following dietary supplements has demonstrated some efficacy in the treatment of depression? A. Glycine B. Melatonin C. Phosphatidylserine D. S-adenosyl-l-methionine (SAMe) E. Nicotinamide adenine dinucleotide (NADH)

D Clinical psychiatry Treatment

Which of the following is the first step in the traditional twelve-step program of addiction recovery? A. Confessing mistakes B. Having faith in a greater power C. Taking a moral inventory of oneself D. Admitting powerlessness over the addiction E. Making amends with those one has harmed

D Clinical psychiatry Treatment

Which of the following is the most commonly used cognitive therapy technique to uncover and modify automatic negative thoughts? A. Exposure B. Free association C. Guided imagery D. Socratic questioning E. Motivational interviewing

D Clinical psychiatry Treatment

Which of the following medications decreases the efficacy of oral contraceptives? A. Bupropion B. Lithium C. Lorazepam D. Oxcarbazepine E. Fluoxetine

D Clinical psychiatry Treatment

A 68-year-old patient recently had a stroke and now exhibits neglect of the left side. What region of the brain was most likely affected? A Left frontal lobe B Right frontal lobe C Left parietal lobe D Right parietal lobe E Right temporal lobe

D Neuroscience

109. Which of the following medications is most likely to precipitate a manic switch in a patient with bipolar I disorder? A. Sertraline B. Phenelzine C. Bupropion D. Mirtazapine E. Desipramine

E

105. A child with intellectual disability is free of childhood seizures but develops epilepsy in adolescence. This pattern of onsct is often seen is which of the following neurodevelopmental disabilities? A. Down syndrome B. Tuberous sclerosis C. Angelman syndrome D. Prader-Willi syndrome E. Autism spectrum disorder (ASD)

E

Which of the following is a methodologic limitation of epidemiologic cohort studies? A. Recall bias B. Ascertainment bias C. Reverse causality D. Control selection E. Loss to follow-up

E Clinical psychiatry Epidemiology

A 65-year-old patient recently began to experience visual hallucinations of children playing. The hallucinations are fully formed, colorful and vivid, but without sound. The patient is not scared or disturbed by their presence, but rather amused. On examination, the patient has normal language and memory, normal cranial nerves, no weakness or involuntary movements and no sensory deficits. Deep tendon reflexes are symmetric. The results of cerebrospinal fluid (CSF) and routine laboratory tests, including toxic drugs screen, are normal. Which of the following is the most likely diagnosis? A. Advanced Alzheimer dementia B. Late onset schizophrenia C. Advanced dementia with Lewy bodies D. Progressive supranuclear palsy E. Posterior cerebral artery ischemia

E Clinical Neurology

A 72-year-old patient is admitted with renal failure. Over the course of several days the patient develops flaccid paralysis, difficulty speaking, dysphagia and impaired eye movements. Correction of which of the following electrolyte deficiencies can be associated with this disorder? A. Potassium B. Magnesium C. Chloride D. Calcium E. Sodium

E Clinical neurology

A 75-year-old patient with a history of extensive tobacco smoking presents with subacute, progressive ataxic disorder over several months, Magnetic resonance Imaging (MRI) scan of the brain is unremarkable. Cerebrospinal fluid shows mild pleocytosis, As part of an extensive evaluation, the patient is found to have a positive anti-Yo antibody. Which of the following tests should be ordered next? A. Colonoscopy B. Genetic testing C. Lumbar puncture D. Electroencephalogram (EEG) E. Computed tomography (CT) scan of the chest

E Clinical neurology

Which of the following is a first-line treatment for restless legs syndrome? A. Carbamazepine B. Clonazepam C. Fluoxetine D. Oxycodone E. Pramipexole

E Clinical neurology

Tracking patient satisfaction for accrediting agencies is characterized as which of the following types of administrative activity? A. Malpractice litigation B. Critical incident review C. Risk assessment review D. Ongoing utilization review E. Continuous quality improvement

E Clinical psychiatry Administration & systems

A child displays appropriate behavior in order to avoid time outs. Which of the following features of operant conditioning is represented? A Extinction B Punishment C Sensitization D Positive reinforcement E Negative reinforcement

E Clinical psychiatry Behavioral & social sciences

Collaborative care interventions for psychiatric disorders in primary care settings include which of the following as one of its core components? A. Location of the psychiatrist in the clinic for consultations B. Collaboration with an outside psychiatric clinic for psychotherapy C. Involvement of the psychiatrist as a member of the primary care treatment team D. Presence of a psychiatric clinic in the immediate vicinity of the primary care clinic E. Care coordination and care management for patients by a non-physician provider

E Clinical psychiatry Consultation/collaborative integrated care

A 10-month-old infant is playing with a ball. When the ball is hidden under a blanket, the child moves the blanket and finds the ball. Which of the following aspects of cognitive development is reflected? A. Seriation B. Assimilation C. Conservation D. Accommodation E. Object permanence

E Clinical psychiatry Development & maturation

According to the predominant theories of adult development, a psychological task of mid-life is to: A. conduct a life review and reflect on past events. B. develop and maintain the capacity for intimacy. C. become a parent and transition to a position of authority. D. establish an adult work identity that becomes part of core identity. E. recognize the limitations of personal time and accept the inevitability of death.

E Clinical psychiatry Development & maturation

Infants appear to be more likely to develop an insecure attachment when they experience poor quality child care if they also have primary caretakers who: A. are working full time. B. are adoptive parents. C. are inexperienced in child care. D. differ in temperament from their child. E. are unresponsive to their infant's needs.

E Clinical psychiatry Development & maturation

A 44-year-old patient presents with extreme fatigue and depression. Physical examination reveals darkening of skin and mucosa. Which of the following tests would most likely lead to the diagnosis? A. Porphobilinogen B. Vanillylmandelic acid (VMA) C. Thyroid-stimulating hormone (TSH) D 5-hydroxyindoleacetic acid (5-HIAA) E. Adrenocorticotropic hormone (ACTH) stimulation test

E Clinical psychiatry Diagnostic procedures

A patient with major depressive disorder is infrequently seen by the psychiatrist because the patient's depressive symptoms have responded very well to antidepressant therapy. The patient continues to see a social worker in an unaffiliated clinic for weekly therapy. The patient is fired from work and reports to the therapist feeling very depressed. This information is not conveyed to the psychiatrist, whom the patient has not seen in two months. The patient then dies by suicide, and the family sues both providers. Which of the following represents the legal responsibility in this situation? A. Only the psychiatrist may be responsible because the psychiatrist is a medical doctor. B. Only the psychiatrist may be responsible because the psychiatrist made the original diagnosis. C. Only the therapist may be responsible because the therapist saw the patient more recently, D. Only the therapist may be responsible because the therapist was obligated to tell the psychiatrist. E. Both providers may be responsible because both have a duty to know the condition of the patient.

E Clinical psychiatry Issues in practice

Immanuel Kant contributed which of the following to the development of psychiatry? A. Existential treatment concepts B. Theory of conflict in the psyche C. Empirical foundations for diagnosis D. Dialectical thinking and problem solving E. Philosophical foundations for clinical ethics

E Clinical psychiatry Issues in practice

Which of the following is an accurate statement about the Mental Health Parity and Addiction Equity Act of 2008 (MHPAEA)? A. Under MHPAEA, all health care plans musts include mental health and substance abuse benefits. B. Before MHPAEA, there were no federal regulations at all concerning mental health parity, C. MHPAEA applies to insurance plans provided by all employers, regardless of size or type of business. D. MHPAEA requires that all businesses that offer employee health insurance provide plans that cover mental health care. E. MHPAEA stipulates that financial requirements for mental health care be no more restrictive than those for medical/surgical care.

E Clinical psychiatry Issues in practice

Fear of abandonment is a core feature of which of the following disorders? A. Agoraphobia B. Delusional disorder C. Separation anxiety disorder D. Avoidant personality disorder E. Borderline personality disorder

E Clinical psychiatry Psychopathology & associated conditions

Paramedics were called to a party after security had to subdue an individual who was swinging at other party-goers and shouting, "You aren't gonna take me without a fight!" On examination the patient has nystagmus, hypertension, and decreased responsiveness to pain. Which of the following substances is the most likely source of the patient's behavior change? A. LSD B. Cocaine C. Marijuana D. Amphetamines E. Phencyclidine (PCP)

E Clinical psychiatry Psychopathology & associated conditions

Which of the following characteristics of major depression is indicative of greater overall severity? A. Fatigue B. Insomnia C. Weight gain D. Somatic complaints E. Psychomotor disturbance

E Clinical psychiatry Psychopathology & associated conditions

Which of the following is increased following sexual reassignment surgery in patients transitioning from male to female? A. Suicide attempts B. Gender dysphoria C. Sexual dysfunction D. Relational problems E. General health problems

E Clinical psychiatry Psychopathology & associated conditions

Chronotherapy is an appropriate treatment for which of the following disorders? A Narcolepsy B Primary insomnia C Primary hypersomnia D Central sleep apnea E Circadian rhythm sleep disorder

E Clinical psychiatry Treatment

Transcranial magnetic stimulation for the treatment of depression targets which of the following brain regions? A. Thalamus B. Motor cortex C. Basal ganglia D. Parietal cortex E. Prefrontal cortex

E Clinical psychiatry Treatment

Which of the following antipsychotic medications is associated with decreased psychotic symptoms, decreased substance use, and increased abstinence in patients with schizophrenia and addictive disorder? A. Thiothixene B. Haloperidol C. Perphenazine D. Chlorpromazine E. Clozapine

E Clinical psychiatry Treatment

Which of the following mechanisms explains the primary effect of grapefiuit juice on the bioavailability of some drugs? A. Protein binding B. Fecal excretion C. Kidney excretion D. Stomach emptying E. Cytochrome metabolism

E Clinical psychiatry Treatment

Which of the following medications has the greatest potential to contribute to the development of galactorrhea in a patient on risperidone? A. Fluvoxamine B. Gabapentin C. Lorazepam D. Nefazodone E. Fluoxetine

E Clinical psychiatry Treatment

Which of the following statistical methods attempts to address the effects of participants dropping out of a study prior to completion? A. Odds ratio B. Meta-analysis C. Power analysis D. Positive predictive value E. Last observation carried forward

E Clinical psychiatry Research & scholarly literacy

134. Dysfunction of which of the following brain regions is most likely responsible for the perseveration evident in the clock drawing? A. Amygdala B. Hippocampus C. Caudate nucleus D. Precentral gyrus E. Prefrontal cortex

E Neurosciences

The efficacy of case management in the treatment of substance use problems is best established for which of the following outcomes? A. Retention in treatment B. Enhanced quality of life C. Reduced substance use D. Lower rates of HIV infection E. Improved occupational function

A Clinical psychiatry Treatment

17. A 75 yr old patient is admitted for confusion. The patient is disoriented to time and place, easily distracted and unable to recall any of the 3 words after 5 minutes. The patient is somnolent much of the day and is often up at night, repeatedly asking, "Where am I ?". The current symptoms are new and different from baseline mild dementia. Workup for the patient's confusion is negative thus far, and haloperidol is started. After 3 days the patient is much improved, with minimal somnolence. The patient continues to be disoriented and is surprised when informed he is in a hospital. Which of the following is the best pharmacologic treatment approach to take next with this patient? A. Discontinue haloperidol B. Continue haloperidol until the disorientation resolves C. Increase the dose of haloperidol until the disorientation resolves D. Switch from haloperidol to risperidone E. Switch from haloperidol to lorazepam

A

20. For children to be optimally stimulated for learning, parents need to provide toys suitable for the infant's level of development as well as novel situations that the infant can begin to grasp. According to Vygotsky, these objects and interactional experiences should be within the infant's: A. zone of proximal development. B. line of visual perception. C. transitional space. D. reflexive abilities. E. sphere of influence

A

22. Ingestion of which of the following may give a false positive urine toxicology screen for benzodiazepines? A. Sertraline B. Ephedrine C. Quetiapine D. Poppy seeds E. Proton-pump inhibitors

A

Which of the following types of therapies is generally most appropriate for patients who have poor reality testing, primitive and immature defenses, Inadequate affect regulation and/or poor impulse control? A. Supportive B. Interpersonal C. Psychoanalytic D. Short-term psychodynamic E. Acceptance and commitment

A Clinical psychiatry Treatment

A two-year-old child presents for routine physical exam. Which of the following skills is the child likely to be able to demonstrate? A. Kick a ball B. Catch a ball C. Copy a circle D. Ride a tricycle E. Hop on one foot

A Neurosciences

34. A 9 year old boy of normal intelligence is seen for evaluation of behavior problems occurring both at home and at school. Parents and teachers describe a 3 to 4 year pattern of persistent intense irritability and anger with almost daily outbursts of verbal and physical aggression triggered by even minor frustrations. The child previously enjoyed playing with friends and participating in team sports. Recently, however, he has become more socially isolated as peers are wary of his angry outbursts, and he has been asked to leave a team because of explosive behavior. Sleep onset is often delayed by the childs refusal to cooperate with bed time routine. Appetite and energy level are normal. The child's view is the the only problem is that "other kids are mean and aduts are unfair." This childs clinical presentation is most highly correlated with an eventual diagnosis of which of the following disorders by the time he reaches young adulthood? A. Anxiety B. Bipolar I C. Somatic symptom D. Narcissistic personality E. Attention-deficit hyperactivity .

A

50. A 16 yr old adolescent reports, "hearing music at night" and "sometimes even hearing my name." During which of the following stages of sleep should this phenomenon occur if they are hypnagogic hallucinations ? A. N1 B. N2 C. N3 D. REM E. Wakefulness

A

53. During a session the psychiatrist tells a long-time patient, "I am moving to another state and will not be able to see you anymore. This is our last session. I am giving you a one-month supply of medication. Here is the letter documenting the end of our treatment. This concludes our session." The patient independently finds another psych1atrast and stablishes care·without incident. Three months later, the patient sues the previous psychiatrist for damages, citing abandonment. Which of the following is""tfie best·defense against medical malpractice? A. Patient did not suffer an appreciably bad outcome. B. Patient filed outside the time period of the statute of limitations. C. Psychiatrist now lives in another state and is not liable for prior activities. D. Psychiatrist fulfilled the duty to the patient by giving 30 days of medication. E. Psychiatrist provided written notification of termination that the patient signed.

A

A 32-year-old patient presents to the emergency department with three days of recurrent headaches. The pain is described as sudden in onset, predominantly periorbital and frequently awakening the patient from sleep. The episodes last for approximately 40 minutes and spontaneously resolve. On examination the patient appears restless, has mild ptosis of the left eye, mild left miosis, increased lacrimation, nasal congestion and rhinorrhea all ipsilateral to the pain. The examination is otherwise unremarkable. Which of the following treatments is most likely to abort the headache? A. Oxygen B. Ibuprofen C. Topiramate D. Indomethacin E. Dexamethasone

A Clinical neurology

A 55-year-old patient complains of "the worst headache of my life" and is evaluated for subarachnoid hemorrhage. Computed tomography (CT) scan of the brain is normal and does not show any blood. Which of the following is the most appropriate next step to determine the presence of subarachnoid blood? A. Lumbar puncture B. Conventional angiogram C. Outpatient follow up in one week D. Repeat CT scan of the brain in 24 hours E. Magnetic resonance imaging (MRI) scan of the brain

A Clinical neurology

The psychiatrist is asking a patient questions describing symptoms of mania. The patient responds, "that sounds exactly like my cousin," and proceeds to tell a long story about the cousin impulsively traveling to Mexico and exhausting all savings, never actually answering the psychiatrist's request. This is an example of: A. tangentiality. B. circumstantiality. C. flight of ideas. D. pressured speech. E. clang associations.

A Clinical psychiatry Diagnostic procedures

Children in which of the following age groups are at greatest risk for death resulting from child abuse? A. Birth - 3 years B. 4 - 6 years C. 7 - 10 years D. Il - 13 years E. 14 - 18 years

A Clinical psychiatry Epidemiology

Which of the following is an aspect of legal standards for competence? A. The law presumes a person is competent. B. Competence must be assessed before each decision. C. The law presumes a person with dementia is incompetent. D. The law presumes a person with active psychosis is incompetent. E. The law presumes a person with developmental delay is incompetent.

A Clinical psychiatry Issues in practice

A patient presents to the primary care clinic with concern about having a brain tumor after an acquaintance died from brain cancer. The patient denies any headache or other neurologic symptoms. However, the patient reports reading on the internet that brain tumors can be easily missed and that there may not be any obvious symptoms initially. There are no neurologic or cognitive findings on exam and a computed tomography (CT) scan is normal. However, the patient returns a month later insisting that other tests such as a brain positron emission tomography (PET) scan be done. The patient has been worried about a missed diagnosis, and has continued to spend hours researching brain tumors. The patient displays no other psychiatric symptoms. Which of the following is the patient's most likely diagnosis? A. Illness anxiety disorder B. Somatic symptom disorder C. Adjustment disorder with anxiety D. Delusional disorder, somatic type E. Functional neurological symptom disorder

A Clinical psychiatry Psychopathology & associated conditions

Which of the following interventions has the most robust evidence for preventing suicide? A. Restricting access to lethal methods B. Media reporting guidelines for suicide C. Public education campaigns about suicide D. Screening interventions within high schools E. Prescription of selective SSRIs for those at high risk

A Clinical psychiatry Psychopathology & associated conditions

10. Which of the following medications has been found to be a secondary cause of restless legs syndrome? A. Opioids B. Antidepressants C. Benzodiazepines D. Dopamine agonists -1 E. Beta adrenergic agonists

B

12. A 30-year-old patient with treatment-resistant schizophrenia is on clozapine. The patient presents to the emergency department with dyspnea, orthostatic hypotension and a heart rate of 130 bpm. Complete blood count is normal except for an elevated eosinophil count. Electrocardiogram (EKG) shows non-specific T-wave changes. Which of the following tests is most likely to be helpful in guiding the acute management of this patient? A. Clozapine level B. Troponin level C. Aldolase level D. Electroencephalogram (EEG) E. Computed tomography (CT) scan of brain

B

15. A 35-year-old construction worker was uninjured when a crane collapsed at a construction site, but witnessed the deaths of two co-workers who were crushed. When evaluated in a hospital emergency department the following day, the patient reports motional numbness, intrusive images, and an inabililty to sleep since the accident. Which of the following interventions IS most appropriate? A. Critical incident debriefing about the work accident B. Normalizing stress reaction and mobilizing relational supports C. Prescribing a benzodiazepine to restabilize the sleep/wake cycle D. Referral for trauma-focused cognitive-behavioral psychotherapy (CBT) E. Prescribing propranolol to decrease long-term risk for chronic posttraumatic stress disorder (PTSD)

B

33. Which of the following antidepressants' plasma levels correlates with therapeutic response? A. Trazodone B. Imipramine C. Amoxapine D. Trimipramine E. Clomipramine

B

46. An 8 yr old child is evaluated for asthma. The pediatrician identifies signficant and ongoing symptoms of ADHD, in the patient as well. Aftter a standard stimulant trial, th parents report little improvement in the child and ongoing oppositional and impulsive behavior at home. While continuing to medically treat the child the pediatrican refers the child to a psychiatrist for medication management, and the parens to a psychologist for prent training. All three professionals monitor the symptoms and communciate regularly. This is an example of what level of collaborative care? A. Primarily primary care B. Shared (integrated) care C. Primarily mental health care D. Primarily primary care with consultation E. Shared (integrated) care and higher levels of care

B

65. Alpha activity in a normal awake adult electroencephalogram (EEG) recording is enhanced by A. drowsiness. B. eye closure. C. hyperventilation. D. photic stimulation. E. use of benzodiazepines.

B

In behavioral psychology, extinction is defined as: A. strengthening behavior by removing an unpleasant event. B. weakening of the conditioned response due to withdrawal of the stimulus. C. diminishing the probability of a behavior by pairing it with an aversive event. D. repeated pairing of a neutral stimulus with one that evokes an emotional response. E. decreasing strength of the association between stimulus and response due to repetition of the stimulus.

B Clinical psychiatry Behavioral & social sciences

A 7-year-old child has an estimated vocabulary of approximately 5000 words, This represents a delay in which of the following domains of language development? A. Grammar B. Semantics C. Pragmatics D. Social communication E. Phonological awareness

B Clinical psychiatry Development & maturation

A pt reports a fear of driving and experiences feelings of panic, SOB, palpitations, sweating, clamminess when anticipating a drive down a steep road. The pt acknowledges going out of the way to avoid this situation. They deny similar symptoms in other settings. Which is the most likely diagnosis? A Panic disorder B Specific phobia C GAD D PTSD E OCD

B Clinical psychiatry Psychopathology & associated conditions

A woman presents to the physician for evaluation of inability to achieve orgasm. Presence of which of the following factors would argue against a diagnosis of female orgasmic disorder? A. Patient concerns about her body image B. Intimate partner violence in current relationship C. Religious beliefs with strong prohibitions against sexual activity D. Discrepancy between patient's and partner's desire for sexual activity E. Ability to achieve orgasm during masturbation but not during partnered sexual activity

B Clinical psychiatry Psychopathology & associated conditions

Children with a tic disorder are less likely than adolescents and adults with a tic disorder to experience co-occurrence of: A. specific learning disability. B. major depressive disorder. C. separation anxiety disorder. D. social anxiety disorder (social phobia). E. attention-deficit/hyperactivity disorder.

B Clinical psychiatry Psychopathology & associated conditions

In order to meet criteria A for schizophrenia, a patient must have at least one of three core symptoms: delusions, hallucinations, or: A. negative symptoms. B. disorganized speech. C. marked decrease in functioning. D. abnormal psychomotor behavior. E. grossly disorganized or catatonic behavior.

B Clinical psychiatry Psychopathology & associated conditions

Which of the following most seriously threatens the external validity of a research study? A. Placebo responses in the study population B. A study population that is not representative of the population to be treated C. Mortality or differential attrition during the course of treatment during the study D. Maturation due to natural change during the period between baseline and post-test E. Statistical regression or regression toward the mean

B Clinical psychiatry Research & Scholarship Literacy

A patient is prescribed fluoxetine 20 mg daily to treat a first depressive episode. After three weeks the patient reports that the medication is "not working" and continues to endorse mild depressive symptoms. Which of the following interventions is most appropriate? A. Discontinue fluoxetine and start citalopram 20mg per day B. Continue fluoxetine 20mg per day and reassess in two weeks C. Continue fluoxetine 20mg per day and add buspirone 5mg twice daily D. Increase the dose of fluoxetine to 40mg per day E. Discontinue fluoxetine and start bupropion 150mg per day

B Clinical psychiatry Treatment

Which of the following interventions is an example of a wraparound, in-home, community-based treatment for youth with behavioral disturbances? A. Parent training B. Multisystemic therapy C. Functional family therapy D. Dialectical behavior therapy E. Parent-child interaction therapy

B Clinical psychiatry Treatment

Research studies have found an association between Alzheimer disease and the accumulation of which of the following proteins inside conical neurons? A. Amyloid B. Tau C. Fibrillin D. Presenilin E. Neuropeptide Y

B Neurosciences

There is robust evidence from epidemiological studies that the offspring of older fathers have an increased risk of which of the following disorders? A. Alcoholism B. Schizophrenia C. Major depression D. Antisocial personality disorder E. Attention-deficit/hyperactivity disorder

B Neurosciences

Which of the following is observed in patients with lesions involving one of the cerebellar hemispheres? A. Gross movements are affected more than fine movements. B. Skilled movements of the extremities ipsilateral to the lesion are disturbed. C. Posture and gait are more severely impaired than for lesions of the vermis. D. Dysmetria and hypotonicity typically affect the leg and foot more than the arm and hand. E. Dysarthria and articulation disturbances are often present and difficult to discriminate from a stroke.

B Neurosciences

42. A patient receives 20$ each time a random urine drug screen is negative for substance use. However, if the urine drug screen is positive, the patient is "locked out" of getting the reward until three consecutive negative urine drug screens are provided. This represents which of the following behavioral techniques? A. Network therapy B. Twelve step facilitation C. Contingency management D. Relapse prevention therapy E. Motivatioal enhancement therapy

C

Prostaglandin D2 helps regulate the sleep-wake cycle by increasing extracellular levels of which of the following chemicals? A. GABA B. Hypocretin C. Adenosine D. Melatonin E. Histamine

C Neurosciences

14. According to DSM 5, for a patient to be diagnosed with insomnia disorder, the sleep difficulty should be present for at least what length of time? A. 1 week B. 1 month C. 3 months D. 6 months E. 1 year

C

26. A 61 year old patient presents to the emergency department ( ED) accompanied by family who report that he patient is unable to remember recent events. The patients memory problems began 2 hours prior, and he patient was congnitively intact before this episode. The patient is alert, slightly anxious and frustrated, and repeatedly asks, "Why am I in the hospital, is some one sick?" This presentation is most consistent with a diagnosis of A. Korsakoff dementia. B. temporal lobe epilepsy. C. transient global amnesia. D. midline cerebellar tumor. E. middle cerebral artery occlusion.

C

36. When a psychiatrist fails to obtain informed consent for treatment, the psychiatrist has failed to respect whfch of the following core principles of medical ethics? A. Justice B. Fidelity C. Autonomy D. Beneficence E. Nonmaleficence

C

38. 38. An 80 yr old patient with major depress1 n has tried previous selective serotonin reuptake inhibitors (SSRis) and one previous serotonin-norepinephrine reuptake inhibitor (SNRI) without significant improvement. The patient has not yet tried a tricyclic antidepressant (TCA). There is no known history of cardiac disease and an electrocardiogram (EKG) is within normal limits. In light of their side effect profiles, which of the following medications is most appropriate for this patient? A. Amitriptyline B. Clomipramine C. Desipramine D. Doxepin E. Imipramine

C

In a community mental health center providing recovery-oriented care, which aspect of the psychiatrists' role is based on recovery-oriented principles? A. Providing trauma-informed care B. Leadership of the multidisciplinary team C. Collaborating in partnership with consumers to support their goals D. Consultation to and supervision of other mental health professions E. Providing diagnostic evaluation and medication management of mental disorders

C Clinical psychiatry Consultation/collaborative integrated care

A child spends several minutes talking with parents recounting how a peer "got in trouble today." The child continues in great detail about how the peer is always breaking the rules. This child is most likely in which of the following age groups? A. Preschool B. Toddlerhood C. School-age D. Early adolescence E. Late adolescence

C Clinical psychiatry Development & maturation

Which of the following characteristics would be most expected of infants who are hypersensitive and less able to tolerate environmental stimuli? A. Lethargic and often withdrawn B. Seldom distressed and often pensive C. Easily startled and more frequent crying D. Recognizing and giving less attention to the familiar E. Inability to discriminate mother's voice from a stranger's

C Clinical psychiatry Development & maturation

A 70-year-old man presents with a 4-month history of worsening cognitive decline, urinary incontinence and difficulty walking. Which of the following are the most likely findings on this patient's structural magnetic resonance imaging (MRI) scan? A. Cortical atrophy B. Cerebellar atrophy C. Enlarged ventricles D. Striatal degeneration E. Diffuse small cortical infarcts

C Clinical psychiatry Diagnostic procedures

The case of "Little Hans" led Freud to develop a psychological theory of the formation of which of the following symptoms? A. Depression B. Regression C. Phobia D. Inattention E. Mania

C Clinical psychiatry Issues in practice

Which of the following is characteristic of persons diagnosed with rapid eye movement(REM) sleep behavior disorder? A. Disorder overwhelmingly affects females B. Episodes are commonly associated with daytime naps C. Polysomnography is notable for REM sleep without atonia D. Episodes occur within the first 60 minutes after sleep onset E. Upon waking from an episode, these individuals are usually confused and disoriented

C Clinical psychiatry Psychopathology & associated conditions

Which of the following is the most common type of dissociative amnesia? A. Failure to recall new events as they occur B. Total loss of memory for one's own personal history C. Failure to recall events during a particular period of time D. Inability to recall some, but not all, events during a particular time E. Inability to recall information in a particular category (e.g., related to one's family)

C Clinical psychiatry Psychopathology & associated conditions

Which of the following psychotherapy modalities is most commonly used in crisis management? A. Psychoanalysis B. Behavioral therapy C. Supportive therapy D. Interpersonal psychotherapy E. Psychodynamic psychotherapy

C Clinical psychiatry Psychopathology & associated conditions

A researcher Investigates whether heavy cannabis use in adolescence is associated with schizophrenia. Daily cannabis use during adolescence in 200 adult participants with schizophrenia is compared with use in 200 participants without a psychiatric diagnosis, Which of the following study designs is the researcher using? A. Cohort study B. Ecological study C. Case-control study D. Cross-sectional study E. Randomized control trial

C Clinical psychiatry Research & scholarship literacy

A patient is referred to psychiatry by the primary care provider (PCP). The PCP informs the psychiatrist that the patient repeatedly complains of abdominal pain and nausea, but an extensive medical work-up was within normal limits. The patient recently immigrated to the United States, and the PCP wonders whether the patient's presentation is a culture-bound syndrome. Asking the patient which of the following questions is most likely to yield information that helps with clinical formulation? A. "Are you homesick?" B. "Why don't you believe the test results?" C. "What do you think is causing your problem?" D, "Do you have trouble expressing your feelings?" E. "Is there something that you are not telling your doctor?"

C Clinical psychiatry Treatment

An interpersonal psychodynamic group comprised of people with heterogeneous diagnoses and levels of functioning has room to take a new member. When considering patients to refer to the group, which of the following diagnoses would be a relative contraindication? A. Bipolar disorder B. Major depression C. Antisocial personality disorder D. Dependent personality disorder E. Narcissistic personality disorder

C Clinical psychiatry Treatment

Pain caused by neurogenic inflammation is mediated by which of the following? A. Metalloproteinase B. Interleukin 6 C. Substance P D. Dopamine E. Complement

C Neurosciences

48. Erikson theorized that a successful developmental task in the 40-to 60-year-old age group is to: A. master intimate relationships despite the demands of work. B. learn to love people without a desire for them to be different. C. become comfortable with' one's identity as perceived by society. D. feel useful to society through behaviors that protect future E. develop a sense of integrity that is able to tolerate the knowledge of impending death.

D

Global mental health most differs from other areas of global medicine in which of the following ways? A. The effects of mental health treatment are simpler to quantify. B. Psychiatry places a greater emphasis on cure rather than care. C. The treatments required can usually occur in a briefer period of time. D. Cultural beliefs have a greater influence on diagnostic understandings. E. There is less demand for psychiatric services than other types of medical care.

D Clinical psychiatry Administration & systems

The clinical observation that depressed adolescents have difficulty with flexible problem-solving has led to studies demonstrating deficits in tests such as the Trails Making Test B. If these findings are sufficiently replicated it would suggest that, for adolescents, a major depressive disorder causes deficits in which of the following domains of cognitive functioning? A Verbal memory B. Working memory C. Processing speed D. Executive functioning E. Visual-spatial memory

D Clinical psychiatry Behavioral & social sciences

Which of the following is a premise of the objects relations theory? A Everyone develops true and false selves B Aggression and envy are central to psychopathology C The "paranoid position" is representative of the first 6 months of life D The mind is inhabited by mental representations of the self and others E The pleasure principle is central to understanding unconscious conflicts

D Clinical psychiatry Behavioral & social sciences

The parents of a four-and-a-half-year-old child report that the child insists there is a small monkey who shares the child's room. The child talks to the monkey, asks others to converse with the monkey and insists that the monkey have a seat at the dinner table. The parents ask if they should be concerned. The best response to the parents is that their child is exhibiting: A. early signs of psychosis. B. signs of visual abnormalities. C. signs of exposure to trauma. D. behavior typical for preschoolers. E. early signs of developmental delay.

D Clinical psychiatry Development & Maturation

A psychiatrist treats a patient in psychotherapy for depression and anxiety. The patient's partner has received a prestigious job offer in another city. Although the patient can easily find work in the other city, the patient feels angry about the possibility of having to move and is thinking of ending the relationship. The patient also reveals feeling a loss of control. According to Erik Erikson, this patient is struggling to navigate which stage of psychosocial development? A. Autonomy versus shame and doubt B. Identity versus role confusion C. Generativity versus stagnation D. Intimacy versus isolation E. Integrity versus despair

D Clinical psychiatry Development & maturation

An 8-year-old child with mild intellectual disability is most likely to have difficulty with which of the following? A. Naming colors B. Following the bedtime routine C. Brushing teeth without supervision D. Understanding social cues of peers E. Accepting hugs from family members

D Clinical psychiatry Psychopathology & associated conditions

For a person to meet diagnostic criteria for enuresis, the behavior must be: A. involuntary in nature. B. during nighttime sleep. C. attributable to a medical condition. D. in a child of at least 5 years of age. E. associated with the effects of a substance.

D Clinical psychiatry Psychopathology & associated conditions

A psychiatrist is conducting a study to assess how a diagnosis of a chronic pain syndrome affects an individual's work performance as measured by the rate of errors. Which of the following statistical measures would be used to describe the probability that an individual with chronic pain will make an error compared to an individual who is pain free? A. Attributable fraction B. Effect size C. Odds ratio D. Relative risk E. Hazard ratio

D Clinical psychiatry Research & scholarship literacy

The main goal of group cognitive behavior therapy for chronic pain treatment is to: A. decrease individual members' pain. B. convince family members that chronic pain is real. C. utilize the group process to avoid thinking about stress. D. improve patients' functioning despite the presence of pain. E. reinforce distorted automatic thoughts about the pain experience,

D Clinical psychiatry Treatment

Which of the following SSRIs has the greatest potential for discontinuation syndrome with missed doses? A. Fluoxetine B. Sertraline C. Citalopram D. Paroxetine E. Escitalopram

D Clinical psychiatry Treatment

Which of the following psychiatric medications should be discontinued prior to ECT? A. Olanzapine B. Fluoxetine C. Memantine D. Lorazepam E. Zolpidem

D Clinical psychiatry Treatment

Which of the following findings would support a diagnosis of ulnar neuropathy at the elbow? A. Decreased sensation in the thumb and second finger and weakness of thumb abduction B. Hypersensitivity to touch in the upper arm and weakness of raising the arm over the head C. Tenderness in the neck radiating to the back of the arm with decreased triceps reflex D. Impaired sensation on dorsum of hand and weakness of finger and wrist extension E. Loss of sensation in the fourth and fifth digit and weakness of intrinsic hand muscles

E Clinical neurology

An unmarried, nulliparous woman participates actively in the rearing of her siblings' children. According to Darwinian theory, such altruistic behavior would be explained by which of the following concepts? A. Sublimation B. Socialization C. Reciprocity D. Maladaptation E. Kin selection

E Clinical psychiatry Behavioral & social sciences

A psychiatrist is consulted regarding a 43-year-old patient who complains of "a lump in my throat," headaches, bloating, back pain, diarrhea, chest pain, painful urination, and sexual indifference. The patient's complaints do not match the objective findings obtained from a thorough diagnostic workup. Which of the following would be best to recommend to the patient's primary care provider? A. Refer to a gastroenterologist B. Refer for intensive psychotherapy C. Prescribe an antipsychotic medication D. Tell the patient there is no medical illness E. Schedule regular visits with a physical examination

E Clinical psychiatry Consultation/collaborative innovative care

Pretend play in preschool-age children depends upon children's increasing ability in which of the following skills? A. Emotion regulation B. Language acquisition C. Cooperation with peers D. Ability to follow parental rules E. Use of one object to represent another

E Clinical psychiatry Development & maturation

Which of the following is a typical phase-of-life problem in middle adulthood? A. Determining one's identity B. Achieving intimacy in relationships C. Choosing a sustainable career path D. Overcoming a sense of invulnerability E. Caring for both one's children and parents

E Clinical psychiatry Development & maturation

A patient presents for evaluation of "sleep problems." Though the patient is not aware of any physical discomfort, the bed partner notices that the patient experiences frequent "jerking" movements in both legs. The patient is referred for a sleep study, which reveals brief bursts of activity on electromyography lasting less than five seconds in duration and occurring every 20-40 seconds. Which of the following is the most likely diagnosis? A. Narcolepsy B. Sleep terror disorder C. Restless leg syndrome D. REM sleep behavior disorder E. Periodic limb movement syndrome

E Clinical psychiatry Diagnostic procedures

A 12-year-old patient is referred for assessment of inattention and behavioral problems in the classroom and the home. Attention-deficit/hyperactivity disorder is diagnosed. The evaluation reveals a significant level of anxiety symptoms. Which of the following recommendations for initial treatment has the strongest evidence base? A. Alpha agonist monotherapy B. Serotonin specific antidepressant monotherapy C. Combined psychostimulant and alpha agonist D. Combined alpha agonist and behavior therapy E. Combined psychostimulant and behavior therapy

E Clinical psychiatry Treatment

A patient with autism is brought to the emergency department due to severe agitation, On examination, the patient has a fever of 38.20 C, heart rate 105, and blood pressure 160/95 mmHg. Leukocyte count is 18.1 103/gL, and creatine kinase is 15000 U/L The patient is noted to be mute, rigid, drooling, and hasn't eaten in 2 days. Chest x-ray and urinalysis are unremarkable. Current medications include fluoxetine, buspirone, methylphenidate, guanfacine, olanzapine, haloperidol, and valproic acid. Which of the following is the most likely diagnosis? A. Malignant catatonia B. Serotonin syndrome C. Anticholinergic toxicity D. Delirium secondary to infection E. Neuroleptic malignant syndrome

E Clinical psychiatry Treatment

A three-year-old girl presents for routine physical examination. Which of the following is the child expected to be able to do? A. Tie shoelaces B. Copy a square C. Copy a triangle D. Have imaginary friends E. Have a sense of herself as female

E Neurosciences

Hearing impairment is a feature of which of the following chromosomal disorders? A. Rett syndrome B. Phenylketonuria C. Wilson disease D. Huntington disease E. Neurofibromatosis-2

E Neurosciences

Which of the following is an example of a retrograde neurotransmitter? A. GABA B. Glutamate C. Dopamine D. Norepinephrine E. Endocannabinoids

E Neurosciences


Related study sets

Chapter 14 Prep-U (harder questions)

View Set

Arbore genealogic si boli autozomale

View Set

Science 1.13 Quiz: Oral Presentation

View Set